1
$\begingroup$
constraints = {A >= 0, B >= 0, A + 2 B == T, S == T*V + 2.4*B^3};
Simplify[Minimize[{T, constraints}, {A, B, T}, Reals], {S > 0, V >= 0}]

Above is the code trying to minimize the time consumption of an item trying to move S.

Its V0=V, Acc0=0.

It is required to perform three stages,

  1. keep V as V0, T=A
  2. Jerk(dS^3/d^3T) = 2.4, T=B
  3. Jerk = -2.4, T=B.

Obviously stage 2&3 are faster than stage 1, so I'm expected to see A->0 in the result. enter image description here But MMA gave me this above. The result of T is same with my solution by hand, but A and B are very weird.

I fully understand the meaning of Roots, and that the result could be expanded with ToRadicals, but that gives me very complex result with complex numbers.

But A was expected to be exactly 0. Other results is an error since I have declared [Reals] in Minimize.

$\endgroup$
1
  • $\begingroup$ By setting S and V to specific numeric value, one can see A is indeed 0. The way to simplify the result is another story, though. (Such simplification can be rather troublesome, see e.g. mathematica.stackexchange.com/q/119346/1871 ) $\endgroup$ Commented Feb 13 at 1:35

0

Your Answer

By clicking “Post Your Answer”, you agree to our terms of service and acknowledge you have read our privacy policy.

Start asking to get answers

Find the answer to your question by asking.

Ask question

Explore related questions

See similar questions with these tags.